Neurosurgery Flashcards

1
Q
  1. A 43-year-old man experiences lower back pain
    after lifting a heavy object off the ground. The
    following morning, he notices that the pain has
    begun to radiate down the posterolateral aspect
    of the right leg and across the top of the foot to
    the big toe. The pain is severe, electric in quality, associated with paresthesia over the same
    distribution, and made worse by coughing. On
    examination, it is found that he has an area of
    diminished sensation to pinprick over the
    dorsum of the right foot and mild weakness in
    his right extensor hallucis longus muscle. The
    deep tendon reflexes are all intact. What is the
    most likely diagnosis?
    (A) Lumbar spinal fracture with compression
    of the cauda equina
    (B) Herniated lumbar disk on the right at
    the level of L4–L5
    (C) Herniated lumbar disk on the left at the
    level of L4–L5
    (D) Herniated lumbar disk on the right at
    the level of S1–S2
    (E) Intermittent claudication
A
  1. (B) The patient has a right-sided L5 radiculopathy, most likely resulting from a disk herniation
    at the right L4–L5 interspace. The key to this
    diagnosis is in understanding the dermatomal
    anatomy of the lower extremity. The L5 dermatomal distribution involves the lateral calf
    and the dorsomedial aspect of the foot. The dermatome also typically includes the big toe.
How well did you know this?
1
Not at all
2
3
4
5
Perfectly
2
Q
  1. A 48-year-old woman has a lower back pain
    and hypoesthesia in the left S1 dermatomal distribution (left calf and lateral left foot). What is
    the most likely cause?
    (A) A lesion at the right L4–L5 interspace
    (B) Pathology where the nerve exits the
    spinal canal immediately above the
    pedicle of S3 vertebra
    (C) A herniated nucleus pulposus
    (D) Compression by the L5 lamina
    (E) A lesion outside the vertebral column
A
  1. (C) Thoracic, lumbar, and sacral nerves exit off
    the spinal canal immediately below the pedicle
    of the corresponding numbered vertebra. The
    left S1 root, for example, passes immediately
    dorsal to the L5–S1 disk, where it can be susceptible to compression by a herniated nucleus
    pulposus. The root then swings laterally to exit
    immediately caudal to the left L5 pedicle. For
    a correlation between level of disk herniation
    and the root affected, see the table below.
    Level of Herniation Root Affected
    L1–L2 L2
    L2–L3 L3
    L3–L4 L4
    L4–L5 L5
    L5–S1 S1
How well did you know this?
1
Not at all
2
3
4
5
Perfectly
3
Q
  1. A 35-year-old secretary complains of severe
    pain in the neck that radiates down the right
    arm. The pain is electric in quality and affects
    specifically the radial aspect of the right forearm
    and the thumb. She also describes numbness
    and paresthesia over the same distribution. On
    physical examination, she is found to have an
    area of diminished sensation to pinprick over
    the right wrist and thumb. The right biceps
    tendon reflex is diminished, but there is no loss
    of muscle strength. She has right C5–C6 disk
    compression and radiculopathy affecting which
    of the following?
    (A) The right C4 root
    (B) The right C4 mixed spinal nerve
    (C) The right C4 anterior primary rami
    (D) The right C6 root
    (E) The right C6 spinal ganglion
A
  1. (D) This patient has radiculopathy of her right
    C6 root. To make this diagnosis, it is essential
    to understand the dermatomal anatomy of the
    upper extremity. The C6 dermatome includes
    the radial aspect of the distal forearm and
    hand. The C4 dermatomes include the deltoid
    region. The biceps tendon jerk is mediated by
    the C5 and C6 roots.
How well did you know this?
1
Not at all
2
3
4
5
Perfectly
4
Q

A 47-year-old man presents to the emergency department after falling from his bicycle. He claims that
his neck was suddenly and violently hyperflexed.
Although he is currently complaining of neck pain,
his chief complaint is weakness of the arms. On
examination, he is found to have profound symmetric weakness of both hands and wrists. His
biceps and triceps are moderately weak. The lower
extremities are only minimally weak, and he is able
to ambulate, albeit with some difficulty. His sensation to all modalities is within normal limits. Plain
radiographs of his neck reveal no fracture or dislocation, but there is evidence of severe spondylosis
with osteophytes narrowing the neural canal at
C3–C4, C4–C5, and C5–C6.

4. What is the most likely mechanism of injury?
(A) Brachial plexus injury
(B) Epidural hematoma
(C) Contusion of the spinal cord
(D) External carotid artery occlusion
(E) Internal jugular vein occlusion
A
  1. (C) The mechanism of injury was a contusion to
    the cervical spinal cord. This probably occurred
    when the violent hyperflexion of the neck
    caused the cervical cord to bump against the
    osteophytic ridges of the spine. The typical clinical picture of a spinal cord contusion is a central cord syndrome.
How well did you know this?
1
Not at all
2
3
4
5
Perfectly
5
Q

A 47-year-old man presents to the emergency department after falling from his bicycle. He claims that
his neck was suddenly and violently hyperflexed.
Although he is currently complaining of neck pain,
his chief complaint is weakness of the arms. On
examination, he is found to have profound symmetric weakness of both hands and wrists. His
biceps and triceps are moderately weak. The lower
extremities are only minimally weak, and he is able
to ambulate, albeit with some difficulty. His sensation to all modalities is within normal limits. Plain
radiographs of his neck reveal no fracture or dislocation, but there is evidence of severe spondylosis
with osteophytes narrowing the neural canal at
C3–C4, C4–C5, and C5–C6.

5. What is this pattern of motor findings that
results from this injury termed?
(A) Central cord syndrome
(B) Cervical radiculopathy
(C) Cauda equina syndrome
(D) Lhermitte sign
(E) Posterior cord syndrome
A
  1. (A) The central spinal cord syndrome describes
    the following pattern of weakness: (a) weakness
    in upper extremity is greater than weakness in
    lower extremity; (b) weakness in distal muscles
    is greater than weakness in proximal muscles
    and limb girdle. This results from the distribution
    of motor fibers within the corticospinal tracts of
    the cervical cord. Fibers supplying the upper
    extremity and more proximal muscles are more
    centrally located and, thus, more susceptible to
    dysfunction from a central injury. Within the
    spinal cord, sensory fibers are more peripherally located and, thus, less frequently affected.
    Sensory deficits, when present, are often variable and inconsistent. A Lhermitte’s sign or syndrome also results from stenosis of the cervical
    canal, causing compression of the spinal cord.
    The patient develops severe numbness and
    paresthesia of the upper extremities as the result
    of sustained hyperextension of the neck.
How well did you know this?
1
Not at all
2
3
4
5
Perfectly
6
Q

A 57-year-old woman is referred to you for evaluation of difficulty with ambulation. Her chief complaint is weakness of her left leg that has been slowly
progressive over the last 6 months. On neurologic
examination, her mental status and cranial nerve
findings are within normal limits. She has marked
(grade 4–5) weakness of both her left leg and arm.
On her left side, she has diminished sensation to
light touch and vibration below the C5 dermatome.
Sensation to pinprick and temperature are severely
diminished on the right side below approximately
the C8 dermatome. Her deep tendon reflexes and
muscle tone are increased on the left.

6. This pattern of neurologic deficits is which of
the following?
(A) Spondylolisthesis
(B) Brown-Sequard syndrome
(C) Central cord syndrome
(D) Guillain-Barré syndrome
(E) Poliomyelitis
A
  1. (B) Brown-Sequard syndrome (Fig. 11–3)
    describe (a) weakness of muscle ipsilaterally
    below the spinal cord lesion, (b) impaired sensation to light touch and vibration ipsilaterally
    below the spinal cord lesion; and (c) impaired
    sensation to pain and temperature contralaterally below the spinal cord lesion.
How well did you know this?
1
Not at all
2
3
4
5
Perfectly
7
Q

A 57-year-old woman is referred to you for evaluation of difficulty with ambulation. Her chief complaint is weakness of her left leg that has been slowly
progressive over the last 6 months. On neurologic
examination, her mental status and cranial nerve
findings are within normal limits. She has marked
(grade 4–5) weakness of both her left leg and arm.
On her left side, she has diminished sensation to
light touch and vibration below the C5 dermatome.
Sensation to pinprick and temperature are severely
diminished on the right side below approximately
the C8 dermatome. Her deep tendon reflexes and
muscle tone are increased on the left.

7. This pattern of neurologic deficits is explained
by injury to the spinal cord with damage to
which of the following?
(A) Anterior horn cells
(B) Peripheral neuropathy
(C) Central cord
(D) Right half (right hemicord)
(E) Left half (left hemicord)
A
  1. (E) The motor deficit is on the left ipsilateral
    side. Brown-Sequard syndrome is caused by
    unilateral injury or dysfunction following
    hemisections of the spinal cord. In the human
    nervous system, motor and sensory functions
    on one side of the body are under the direct
    control of the opposite side of the brain. All
    major motor and sensory tracts decussate. The
    decussation of the various tracts occurs at different levels of the neuraxis.
How well did you know this?
1
Not at all
2
3
4
5
Perfectly
8
Q
  1. A 73-year-old man presents for evaluation of
    weakness in his lower extremities and recurrent
    falls. On further questioning, the patient admits
    to having frequent spasms affecting both of his
    lower extremities. He also claims that his legs
    occasionally feel as if ants were crawling all over
    them. On neurological examination, he is found
    to have a slightly unstable gait and with minimal flexion of the knees. His strength is slightly
    but symmetrically diminished in both lower
    extremities and both triceps muscles. There is
    decreased sensation to vibration and light touch
    below approximately the level of the nipples
    bilaterally. In both lower extremities, muscle
    tone is markedly increased, and deep tendon
    reflexes are hyperactive. Babinski’s reflex is present bilaterally. What is the most likely diagnosis?
(A) A thoracic spinal cord compression
(B) A thoracic radiculopathy
(C) A cervical myelopathy
(D) Cerebellar tumor
(E) Intracranial aneurysm
A
  1. (C) On subsequent MRI of the cervicothoracic
    spine, this patient is found to have severe
    spondylosis at multiple levels of the spine.
    There is spinal cord compression by a large
    osteophyte at the level of C6–C7. The patient
    has all the signs and symptoms of cervical
    spinal cord dysfunction. The weakness affecting the triceps muscles in addition to the lower
    extremities indicates that the lesion is above
    the level of the thoracic cord. Absence of similar symptoms on the face as well as the absence
    of cranial nerve abnormalities indicate that
    the lesion is not intracranial. The diffuseness of
    the symptoms as well as the fact that they are
    associated with increased reflexes and tone
    indicate that the problem lies within the CNS
    (upper motor neuron) rather than the peripheral nervous system (lower motor neuron).
How well did you know this?
1
Not at all
2
3
4
5
Perfectly
9
Q
  1. An 87-year-old woman is referred to you for
    evaluation of lower back pain. It is exacerbated
    by walking or prolonged standing and occasionally made better by bending over. Physical
    examination reveals a thin, elderly woman
    who walks with a cane with her lower back
    moderately flexed. Motor power in her lower
    extremities is normal, but she has impaired
    sensation to light touch and vibration below
    the L4 dermatome bilaterally. Deep tendon
    reflexes are normal in her upper extremities
    but absent in both lower extremities. You refer
    her for magnetic resonance imaging (MRI) of
    the lumbosacral spine. What will be the most
    likely finding on this study?
    (A) Lumbar spinal stenosis
    (B) A fracture of the odontoid process
    (C) A herniated L3–L4 disk causing unilateral
    compression of the L4 root
    (D) Spinal cord compression at the level of
    L1 vertebra level
    (E) Spinal cord compression at the T1
    vertebra level
A
  1. (A) The clinical presentation indicates a lower
    motor neuron lesion. The clinical diagnosis is
    neurologic claudication secondary to lumbar
    spinal stenosis, which is commonly seen in elderly persons in whom (as a consequence of
    wear and tear over the years) bony structures
    of the lumbar spine hypertrophy and develop
    osteophytes. These bony changes, in turn, lead
    to stenosis of the spinal canal and intervertebral
    foramina. Thus, the result is compression and
    dysfunction of multiple lumbosacral nerve
    roots bilaterally. Bending over opens the
    lumbar canal and relieves the stenosis.
How well did you know this?
1
Not at all
2
3
4
5
Perfectly
10
Q
  1. A 33-year-old man is brought to the emergency
    department after being involved in a major
    motor vehicle accident. He is unable to move his
    legs and complains of severe pain in his mid
    to lower back. On physical examination, he is
    found to have exquisite tenderness over some of
    the bony prominence of his lower back, but no
    gross physical deformity can be appreciated. On
    neurologic examination, flaccid paralysis of both
    lower extremities and complete anesthesia to all
    sensory modalities below approximately the L3
    dermatome are noted. Catheterization of his
    bladder yields approximately 700 mL of urine.
    Plain radiographs of the spine reveal compression fracture in the body of L3 with greater than
    50% of loss in its height. A computed tomography (CT) scan through this area reveals a burst
    fracture of the body of L3. There are large fragments of bone driven dorsally with an 80% canal
    compromise. What is the cause of weakness?
    (A) Compression of the conus medullaris
    (B) Compression of the spinal cord at the
    level of L3
    (C) Compression of the cauda equina
    (D) Rupture of the anterior spinal ligament
    (E) Associated epidural hemorrhage
A
  1. (C) This patient has suffered a traumatic fracture of L3 in which bony fragments were displaced dorsally to compress the cauda equina
    at that level. It is important to remember that
    the spinal cord does not extend along the entire
    length of the spine. The conus medullaris, the
    most caudal tip of the spinal cord, ends in 98%
    of people at or above L2 vertebrae. Thus, it is
    highly unlikely for an L3 fracture to cause compression of the spinal cord or conus medullaris.
How well did you know this?
1
Not at all
2
3
4
5
Perfectly
11
Q

A 17-year-old boy suffers a hyperextension injury of his
neck when he jumps headfirst into a shallow pool. He
does not lose consciousness. He arrives at the emergency department holding his neck stiffly and complaining of severe neck pain. He says the pain is
particularly severe whenever he tries to move his head.
He says he has no neurologic symptoms such as weakness, numbness, or paresthesia. On physical examination, he is found to have no areas of ecchymosis or deformity on the cervical spine. He has exquisite pain
on deep palpation of the bony prominence of the midcervical spine. There are no neurological signs. Routine
plain radiographs (anteroposterior [AP], lateral, openmouth view) of the cervical spine in the neutral position show no fracture or subluxation of the bony
elements. There is, however, thickening of the pretracheal space ventral to the body of C6, suggesting softtissue swelling.

  1. What would the next step in management
    involve?
    (A) Analgesics alone
    (B) A hard cervical collar
    (C) Internal fixation of the cervical vertebra
    (D) Burr holes and traction
    (E) Plaster cast to face, neck, and thorax
A
  1. (B) The most appropriate step is to place him in
    a hard cervical collar to protect his neck and
    obtain plain lateral radiographs in flexion and
    extension. In this boy, the continuous neck pain
    and the prevertebral swelling on the plain radiographs are strongly suggestive of an injury to
    the ligamentous structures of the cervical spine.
    A severe ligamentous tear can lead to instability of the spine from excessive movement
    between adjacent vertebrae. Ligamentous
    injury must be ruled out by obtaining lateral
    radiographs in flexion and extension to demonstrate any excessive movement between adjacent vertebrae. This excessive movement, if
    missed, can result in compression of the cervical spinal cord and a serious neurologic deficit.
    These studies require supervision by appropriate specialist consultants.
How well did you know this?
1
Not at all
2
3
4
5
Perfectly
12
Q

A 17-year-old boy suffers a hyperextension injury of his
neck when he jumps headfirst into a shallow pool. He
does not lose consciousness. He arrives at the emergency department holding his neck stiffly and complaining of severe neck pain. He says the pain is
particularly severe whenever he tries to move his head.
He says he has no neurologic symptoms such as weakness, numbness, or paresthesia. On physical examination, he is found to have no areas of ecchymosis or deformity on the cervical spine. He has exquisite pain
on deep palpation of the bony prominence of the midcervical spine. There are no neurological signs. Routine
plain radiographs (anteroposterior [AP], lateral, openmouth view) of the cervical spine in the neutral position show no fracture or subluxation of the bony
elements. There is, however, thickening of the pretracheal space ventral to the body of C6, suggesting softtissue swelling.

  1. What would be the most appropriate radiologic examination?
    (A) Plain lateral radiographs in flexion and
    extension to rule out occult ligamentous
    tear and instability of the cervical spine
    (B) A CT scan of the cervical spine to rule
    out the possibility of a bony fracture not
    seen on plain radiographs
    (C) Lateral tomogram of the cervical spine
    to rule out the possibility of an occult
    fracture
    (D) Angiography
    (E) Ultrasound of the neck
A
  1. (A) A CT scan of the cervical spine is more sensitive for fractures of the spine than are plain
    Answers: 1–12 251
    Figure 11–3.
    Brown-Sequard syndrome. The lesion depicted here is at a lower
    spinal cord level than that described in the text. (Reproduced, with
    permission, from Lindner HH: Clinical Anatomy. Appleton &
    Lange, 1989.)
    radiographs. Because CT images are in the axial
    plane, only one vertebral body can be seen at a
    time. This makes CT scanning entirely inadequate to rule out all but large subluxation
    resulting from the most major ligamentous disruptions. Sagittal MRI of the cervical spine in
    this case may show swelling or hematoma
    within the soft tissues of the spine. MRI, however, is poor in demonstrating bony anatomy
    and detail. Furthermore, without flexion and
    extension of the neck, an MRI of the cervical
    spine is no better in showing bony instability
    than plain radiographs in the neutral position.
How well did you know this?
1
Not at all
2
3
4
5
Perfectly
13
Q

A 63-year-old woman with a history of local inoperable breast cancer is referred to you for the evaluation of
new-onset diplopia. Upon questioning, she admits that
diplopia occurs mostly when she attempts to look at
objects in the distance and when she attempts to look
toward the left side. In addition, she reports having
severe headaches and an electric-type discomfort
affecting her right deltoid region for approximately
3 weeks. On neurologic examination, she is found to
have left abducens (sixth) nerve palsy; the rest of her
cranial nerves are intact. She also has mild weakness of
the right deltoid and a diminished biceps tendon jerk
on the same side. Findings on an MRI of the brain with
intravenous contrast are unremarkable.

13. In this patient, what would be the most likely
site where metastasis occurs?
(A) Brain
(B) Orbital cavity
(C) Meninges
(D) Cerebellum
(E) Optic chiasm
A
  1. (C) Meningeal carcinomatosis results when
    malignant cells gain access to the CSF and are
    able to disseminate within it. Cells most commonly adhere to and affect the neural structures traversing the CSF, such as cranial nerves
    and peripheral nerve roots. Cells cause dysfunction at multiple sites of the CNS. This
    patient has a left abducens nerve palsy and a
    right C5 radiculopathy, making the diagnosis of
    meningeal carcinomatosis highly likely.
How well did you know this?
1
Not at all
2
3
4
5
Perfectly
14
Q

A 63-year-old woman with a history of local inoperable breast cancer is referred to you for the evaluation of
new-onset diplopia. Upon questioning, she admits that
diplopia occurs mostly when she attempts to look at
objects in the distance and when she attempts to look
toward the left side. In addition, she reports having
severe headaches and an electric-type discomfort
affecting her right deltoid region for approximately
3 weeks. On neurologic examination, she is found to
have left abducens (sixth) nerve palsy; the rest of her
cranial nerves are intact. She also has mild weakness of
the right deltoid and a diminished biceps tendon jerk
on the same side. Findings on an MRI of the brain with
intravenous contrast are unremarkable.

  1. What would the next step in management
    involve?
    (A) An MRI of the cervical spine to rule out
    metastatic deposits within the cervical
    roots
    (B) A CT scan of the brain with intravenous
    contrast
    (C) A lumbar puncture to measure opening
    pressure and obtain cerebrospinal fluid
    (CSF) for cytologic analysis
    (D) Repeated breast biopsy
    (E) No further tests until further symptoms
    develop
A
  1. (C) In the presence of meningeal carcinomatosis (also called carcinomatous meningitis), the
    lumbar puncture CSF examination may reveal
    elevated protein and positive cytology. The sensitivity of MRI to detect small tumor deposits
    within the intracranial compartment is much
    greater than that of a CT scan. Thus, a CT scan
    is unlikely to be helpful in this clinical scenario.
How well did you know this?
1
Not at all
2
3
4
5
Perfectly
15
Q
  1. A 57-year-old woman presents to the emergency
    department with new-onset seizures. She was
    witnessed by her husband to have a generalized seizure lasting approximately 1 minute. She
    has smoked 1 pack of cigarettes a day for over
    40 years. In the past 3 months, she has lost 25 lb
    in weight. On examination, she appears thin
    and nervous but findings on her neurologic
    examination are otherwise essentially within
    normal limits. Plain radiographs of the chest
    obtained in the emergency department show a
    4-cm nodule in the upper lobe of her right lung.
    To exclude cerebral metastasis as a cause of her
    seizure, what should the next test requested be?
    (A) An electroencephalogram (EEG)
    (B) A CT scan of the brain with intravenous
    contrast
    (C) A spinal tap to measure opening pressure
    and obtain CSF for cytology
    (D) An MRI of the brain with intravenous
    contrast
    (E) Doppler ultrasound
A
  1. (D) An adult with new onset seizures is considered to have a brain tumor until proved otherwise. The best test available to detect metastatic
    deposits in the brain is the MRI with intravenous
    contrast. MRI is exquisitely sensitive in diagnosing brain metastasis, sometimes detecting
    them by the brain edema they induce even when
    the lesion itself is too small to be seen. The EEG
    may likely show the presence of seizure activity
    and even localize it to a particular region of the
    brain; it will not, however, answer the question
    of what pathologic process is responsible. Also,
    in this case, because a mass lesion is expected,
    performing a spinal tap is relatively contraindicated for the fear of inducing uncal herniation in
    a patient who may have increased ICP.
How well did you know this?
1
Not at all
2
3
4
5
Perfectly
16
Q

A 58-year-old woman is admitted from the emergency
department with a history of approximately 2 weeks
of headache. She has a history of breast cancer. Her
headache is severe, particularly in the mornings when
she wakes up. It is accompanied by occasional vomiting. She says she experiences no focal weakness,
numbness, or paresthesia. On physical examination,
she is found to have a mild weakness of her left arm.
An MRI of the brain with intravenous contrast reveals
the presence of a neoplasm in the right motor cortex
that is considered responsible for her weakness.

16. If the MRI shows multiple brain metastasis,
what should be the treatment required in addition to corticosteroids?
(A) Whole-brain radiotherapy
(B) Craniotomy to resect the lesion
responsible for her left arm weakness
(C) Chemotherapy
(D) Placement of an Ommaya reservoir for
use in treatment by intrathecal
chemotherapy
(E) No further treatment
A
  1. (A) The optimal management of any intracranial
    neoplasm includes use of corticosteroids. These
    significantly diminish the amount of tumorinduced brain edema and are remarkably effective in ameliorating symptoms caused by CNS
    neoplasms. The current recommendation for the
    treatment of multiple brain metastasis is treatment with a full course of fractionated radiation to the whole brain. This is geared to treat all
    visible lesions within the parenchyma as well as
    those that may still be too small to be detected.
    Intrathecal chemotherapy is effective in treating meningeal carcinomatosis, where the primary site of involvement is the meninges and
    the surface of the brain. The two available agents
    for this modality of treatment have very poor
    penetration into deeper regions of the brain
    when administered intrathecally.
How well did you know this?
1
Not at all
2
3
4
5
Perfectly
17
Q

A 58-year-old woman is admitted from the emergency
department with a history of approximately 2 weeks
of headache. She has a history of breast cancer. Her
headache is severe, particularly in the mornings when
she wakes up. It is accompanied by occasional vomiting. She says she experiences no focal weakness,
numbness, or paresthesia. On physical examination,
she is found to have a mild weakness of her left arm.
An MRI of the brain with intravenous contrast reveals
the presence of a neoplasm in the right motor cortex
that is considered responsible for her weakness.

17. If the MRI shows a single brain metastasis,
what should be the next step in management?
(A) Whole-brain radiotherapy
(B) Craniotomy to resect the lesion
responsible for her left arm weakness
(C) Chemotherapy
240 11: Neurosurgery
(D) Placement of an Ommaya reservoir for
use in treatment by intrathecal
chemotherapy
(E) No further treatment
A
  1. (B) Surgical resection is recommended only for
    cases involving a single brain metastasis that is
    surgically accessible in patients with a reasonable life expectancy. It is also relatively indicated in patients with multiple brain lesions in
    whom one particular lesion is imminently lifethreatening. Intravenous chemotherapy has,
    unfortunately, yielded poor results in the
    treatment of brain metastasis. This is particularly so in this patient, because her tumors are
    already likely to be resistant to the chemotherapeutic agents with which she has already been
    treated.
How well did you know this?
1
Not at all
2
3
4
5
Perfectly
18
Q
  1. A 63-year-old woman presents with a severalweek history of headaches and difficulties with
    speech. A sister who lives with her claims that
    her language “has recently not been making
    much sense” and that she is a bit confused. Her
    condition seems to be deteriorating. On neurologic examination, she has a moderately
    severe aphasia, with difficulty understanding
    language and following commands, and she
    makes frequent paraphasic errors when she
    speaks. There are no other motor or sensory
    deficits. An MRI with intravenous contrast
    reveals the presence of a ring-enhancing mass
    lesion within the substance of the left temporal
    lobe. The lesion is approximately 3 cm in greatest diameter, poorly demarcated from the surrounding brain, and surrounded by a moderate
    amount of cerebral edema. Findings on routine admission tests, including a chest x-ray
    and serum chemistry, are unremarkable. What
    is the most likely diagnosis?
    (A) Low-grade cerebral astrocytoma
    (B) Glioblastoma multiforme
    (C) Metastasis to the brain from an occult
    primary cancer
    (D) Meningioma
    (E) Glomus tumor
A
  1. (B) Glioblastoma multiforme is a highly malignant neoplasm, arising from glial cells or their
    precursors within the CNS. It is the most
    common of all primary malignancies of the CNS
    and its peak incidence is within the fifth to seventh decade of life. A low-grade astrocytoma is
    a tumor derived from glial cells of astrocytes.
    Fig. 11–4, shows a large cystic giant astrocytoma
    on T2 weighted MRI where fluid is shown as a
    white area with midline shift (not glioblastoma
    multiforme presented in this question).
How well did you know this?
1
Not at all
2
3
4
5
Perfectly
19
Q
  1. A 64-year-old man presents with headache and
    left-sided upper extremity weakness. The MRI
    findings suggest that this is a glioblastoma
    multiforme. This is because the tumor exhibits
    which of the following?
    (A) It is regular in shape.
    (B) It is well demarcated from surrounding
    brain tissue.
    (C) It shows a ring pattern of enhancement
    with intravenous contrast and has a
    nonenhancing necrotic center.
    (D) It shows an absence of surrounding
    white-matter edema.
    (E) It arises from the carotid body.
A
  1. (C) Glioblastoma multiforme grows rapidly, and
    the tumor often contains a necrotic core that
    occurs as its growth surpasses its blood supply.
    Additional features on MRI include irregular
    shape, poor demarcation from surrounding
    252 11: Neurosurgery
    brain tissue, and the presence of variable
    amount of surrounding white-matter edema.
How well did you know this?
1
Not at all
2
3
4
5
Perfectly
20
Q
  1. A 63-year-old woman presents for workup to
    determine the reason for a gradual hearing loss
    over approximately 5 years and intermittent
    tinnitus over the last several months. Findings
    on physical and neurologic examination are
    entirely within normal limits, except for the
    presence of sensorineural hearing loss in the
    left ear. She has no cranial nerve deficits. An
    MRI of the brain with gadolinium reveals the
    presence of an extra-axial tumor in the region
    of the left cerebella-pontine angle. What is the
    most likely diagnosis?
    (A) Epidermoid tumor (cholesteatoma)
    (B) Glioblastoma multiforme
    (C) Meningioma
    (D) Acoustic neuroma
    (E) Glomus tumor
A
  1. (D) This cerebella-pontine angle tumor is most
    likely an acoustic neuroma. This is the most
    commonly encountered neoplasm in this
    region. It arises from the Schwann cells that
    form the myelin sheath of the vestibular division of the eighth cranial nerve (hence a more
    accurate name is vestibular schwannoma). This
    tumor typically arises within the internal
    acoustic canal and growths in the direction of
    least resistance—through the meatus into the
    cerebellopontine angle cistern.
How well did you know this?
1
Not at all
2
3
4
5
Perfectly
21
Q

A 4-year-old boy is brought to the emergency department with the complaint of approximately 2 weeks of
headache and vomiting. He was seen in the emergency department 1 week earlier with the same complaints. At that time, his parents were told that the
probable cause was a gastrointestinal virus, and
the boy was sent home. His symptoms have not
improved. On general examination, the child appears
somewhat dehydrated and has a dry mouth and
sunken eyes. His examination findings are also
remarkable for the presence of bilateral papilledema
and marked nystagmus. An MRI with intravenous
contrast is obtained that reveals the presence of a
2-cm mass in the posterior fossa. The mass is entirely
within the fourth ventricle and appears to be arising
from the vermis of the cerebellum. It enhances uniformly with contrast. The lateral and third ventricles
are moderately dilated with hydrocephalus.

21. What is the most likely diagnosis?
(A) Acoustic neuroma
(B) Craniopharyngioma
(C) Medulloblastoma
(D) Brain metastasis
(E) Polycystic cerebellar astrocytoma
A
  1. (C) An astute neurologist once said that in neurologic diagnosis, as in real estate, location is
    everything. He alluded to the fact that in the
    diagnosis of neurologic ailments, one can often
    generate lists of possible diagnoses based solely
    on the location of the lesion in question. With
    unusual exceptions, each location within the
    CNS is likely to be associated with a certain type
    of neoplasm. The medulloblastoma (also called
    a primitive neuroectodermal tumor or PNET) is
    a highly aggressive and rapidly growing tumor
    that most often arises within the cerebellar
    vermis. It usually grows locally as a roughly
    spherical mass to bulge into and obliterate
    the adjacent fourth ventricle. Ependymoma or
    choroid plexus papilloma should also be considered in the differential diagnosis.
How well did you know this?
1
Not at all
2
3
4
5
Perfectly
22
Q

A 4-year-old boy is brought to the emergency department with the complaint of approximately 2 weeks of
headache and vomiting. He was seen in the emergency department 1 week earlier with the same complaints. At that time, his parents were told that the
probable cause was a gastrointestinal virus, and
the boy was sent home. His symptoms have not
improved. On general examination, the child appears
somewhat dehydrated and has a dry mouth and
sunken eyes. His examination findings are also
remarkable for the presence of bilateral papilledema
and marked nystagmus. An MRI with intravenous
contrast is obtained that reveals the presence of a
2-cm mass in the posterior fossa. The mass is entirely
within the fourth ventricle and appears to be arising
from the vermis of the cerebellum. It enhances uniformly with contrast. The lateral and third ventricles
are moderately dilated with hydrocephalus.

  1. If at craniotomy the tumor found is not that
    listed in question 21 and the pathologist reports
    that it is a benign lesion, what is that lesion?
    (A) Ependymoma
    (B) Choroid plexus papilloma
    (C) Polycystic (cystic) cerebellar astrocytoma
    (D) Teratoma
    (E) Dermoid cyst
A
  1. (B) Choroid plexus papillomas are benign
    tumors of the CNS that arise from the cells that
    form the choroid plexus. These tumors can be
    found wherever choroid plexus is present,
    including the lateral and fourth ventricles.
    They cause symptoms of increased ICP, most
    commonly by causing massive degrees of
    hydrocephalus. This can be from two mechanisms—obstruction of normal CSF pathways or
    production by the tumor of excessive volumes
    of CSF. (Remember that CSF is produced
    mainly by the choroid plexus.) Ependymomas
    are also highly malignant tumors usually
    found in the fourth ventricle of children. Its
    precursor cell is the ependymal cell that lines
    the ventricular system. As medulloblastomas,
    these tumors are highly aggressive and fast
    growing. Contrary to the former, however,
    ependymomas tend to arise from the floor of
    the fourth ventricle (the dorsal surface of the
    brainstem).
How well did you know this?
1
Not at all
2
3
4
5
Perfectly
23
Q

A 5-year-old girl undergoes debulking of medulloblastoma. She undergoes a repeat MRI of the brain
with intravenous contrast, which shows a small
amount of enhancement consistent with limited residual tumor. She is given a full course of radiotherapy
to the posterior fossa and does very well for 6 weeks,
until she experiences difficulty in walking. Physical
examination at this time indicates moderate weakness of both lower extremities (particularly on the
right side) but strength in her upper extremities and
cranial nerves are normal. Her sensation to light touch
and vibration are intact, but she has diminished sensation to pinprick throughout her left leg.

  1. What should be the next step in management?
    (A) Repeat the MRI of the brain to rule out
    an early recurrence
    (B) Obtain a single-photon-emission CT
    (SPECT) scan of the brain to rule out the
    possibility of radiation-induced toxicity
    (C) Begin treatment with chemotherapy for
    the residual tumor within the brain
    (D) Obtain an MRI or myelogram of the
    entire spinal axis to rule out the
    possibility of “drop metastasis” from the
    medulloblastoma
    (E) Obtain an ultrasound of the lumbar spine
A
  1. (D) Obtain an MRI or myelogram of the entire
    spinal axis to rule out the possibility of “drop
    metastasis” from the medulloblastoma. The
    constellation of emerging new symptoms
    points toward spinal cord dysfunction; the
    Answers: 13–23 253
    Figure 11–4.
    Large cystic giant astrocytoma on T2 weighted MRI where fluid is shown as a white area. Midline shift.
    most likely cause is the presence of drop metastasis from the medulloblastoma. Primary CNS
    neoplasms rarely metastasize outside of their
    site of origin. Exceptions to this statement
    include both medulloblastoma and ependymoma. These tumors shed viable cells into the
    CSF, where they are transferred to such distant
    areas as the intracranial or, more commonly,
    the spinal subarachnoid space. There they can
    lodge and replicate to form tumor nodules that
    can compress adjacent neural structures. The
    test of choice for diagnosing the presence of
    these drop metastasis is a MRI of the spine with
    intravenous contrast or a myelogram.
How well did you know this?
1
Not at all
2
3
4
5
Perfectly
24
Q

A 5-year-old girl undergoes debulking of medulloblastoma. She undergoes a repeat MRI of the brain
with intravenous contrast, which shows a small
amount of enhancement consistent with limited residual tumor. She is given a full course of radiotherapy
to the posterior fossa and does very well for 6 weeks,
until she experiences difficulty in walking. Physical
examination at this time indicates moderate weakness of both lower extremities (particularly on the
right side) but strength in her upper extremities and
cranial nerves are normal. Her sensation to light touch
and vibration are intact, but she has diminished sensation to pinprick throughout her left leg.

  1. What should treatment of this girl involve?
    (A) Removal of recurrent medulloblastoma
    and neck dissection
    (B) Ventriculoperitoneal shunt
    (C) Repeat irradiation to the posterior cranial
    fossa
    (D) Complete craniospinal irradiation with
    local boosts to the areas where tumor
    nodules are detected
    (E) Cortisone alone
A
  1. (D) Treatment of drop metastasis consists primarily of complete craniospinal irradiation with
    local boosts to the areas where tumor nodules
    are detected. Chemotherapy, particularly a combination of procarbazine, lomustine (CCNU),
    and vincristine (PCV), is usually given to treat
    disease that is locally recurrent after maximal
    irradiation. Radiation-induced toxicity or radionecrosis is highly unlikely to be the cause of
    these newly developed symptoms. The first
    reason for this is that the child’s new symptoms
    and findings appear to be exclusively spinal in
    origin. Second, radiation-induced necrosis, a
    feared complication of CNS irradiation, is never
    observed in such a short interval after completing treatment.
How well did you know this?
1
Not at all
2
3
4
5
Perfectly
25
Q
  1. A 35-year-old man is brought to the hospital
    unconscious after being resuscitated in an ambulance from the site of a motor vehicle accident.
    No other history or information is available.
    On general inspection, he is found to have multiple bruises over his body and has a massively
    swollen left thigh. His vital signs are stable with
    a heart rate of 100 beats per minute (bpm) and
    a blood pressure of 150/75 mm Hg. He is
    obtunded and does not follow commands or
    open his eyes. He withdraws his left arm and
    leg from painful stimuli, but not his right. His
    left pupil is 3 mm in diameter, and it is sluggishly reactive to light, while his right is 5 mm
    in diameter and fixed. Corneal reflexes are present bilaterally. His pulse rate is 120 bpm and respiration rate is 40 breaths per minute. To avoid
    injury to his spinal cord by an unstable cervical
    spine, an order is issued to not perform testing
    of his doll’s eye reflex. Intracranial hemorrhage
    causing increased intracranial pressure (ICP) is
    suspected, along with a right uncal herniation.
    What is the next step in management?
    (A) Intubation of his airway for
    hyperventilation and administration of
    intravenous mannitol
    (B) Immediate CT scanning of the brain to
    confirm the presence of the suspected
    intracranial hemorrhage
    (C) Intubation of his airway for
    hyperventilation and intravenous
    administration of corticosteroids
    (D) Immediately evacuation of the suspected
    intracranial hematoma
    (E) Controlled hypoventilation
A
  1. (A) Intubation will accomplish two purposes.
    First, it will protect the airway and prevent
    the possibility of aspiration. Second, it will
    allow controlled hyperventilation (PCO2 of
    25–30 mm Hg), which causes cerebral vasoconstriction, which, in turn, transiently lowers
    ICP and reduces intracranial intravascular
    blood volume. Mannitol will reduce intracerebral pressure and volume. The role of corticosteroids in the management of cerebral
    trauma is controversial at best. Their advocates propose that corticosteroids work by
    reducing the amount of traumatically induced
    brain edema. Even these investigators concur
    that their effect is not immediate and that they
    take at least 4–6 hours to work. The subdural
    space is between the inner layer of dura and
    the arachnoid.
How well did you know this?
1
Not at all
2
3
4
5
Perfectly
26
Q
26. In the management of a 64-year-old woman
struck by a car, mannitol is given to do which
of the following?
(A) Increase CSF formation
(B) Increase the respiratory rate
242 11: Neurosurgery
(C) Increase the pulse rate
(D) Replace extensive fluid loss
(E) Lower raised ICP
A
  1. (E) Mannitol is a complex sugar that remains in
    the intravascular space because of its high
    molecular weight. When it is given in large
    doses (1–2 g/kg of body weight), water is
    extracted from the cerebral interstitium by its
    osmotic effect, causing reduction in total brain
    volume. Both these measures are temporizing
    steps to allow enough time for definitive diagnosis and treatment to take place. The effect of
    hyperventilation on ICP rapidly wears off after
    a few hours. Over time, mannitol will diffuse
    into the cerebral interstitium, losing its effectiveness and even exacerbating cerebral edema.
    A note of caution, however, mannitol is an
    osmotic diuretic and as such must be given
    with extreme caution in the setting of hypotension due to excessive blood loss.
How well did you know this?
1
Not at all
2
3
4
5
Perfectly
27
Q
  1. A 17-year-old boy is brought to the emergency
    department after he was assaulted. Witnesses
    claim that he was hit on the head with a lead
    pipe, after which he was unconscious for several minutes. No seizure activity was witnessed. On arrival, he complains of a headache,
    particularly severe at the point where he was
    hit in the right frontoparietal region. On examination, he is found to have swelling and ecchymosis over this region. He is awake, alert, and
    fully oriented. A complete neurologic examination reveals no deficit. Plain radiographs of
    the skull show a linear, nondepressed skull
    fracture in the frontoparietal skull that crosses
    the groove of the medial meningeal artery.
    During the following hour, he becomes sleepier
    and begins to vomit. A repeat neurologic examination at that time reveals him to be lethargic
    but without weakness, numbness, paresthesia,
    or other focal deficit. What is the most likely
    cause of the neurologic deterioration?
    (A) Diffuse axonal injury (DAI)
    (B) Todd’s phenomenon
    (C) Subdural hematoma
    (D) Epidural hematoma
    (E) Trigeminal ganglion hematoma
A
  1. (D) This is the classic presentation of an acute
    epidural hematoma (Fig. 11–5) transient traumatic loss of consciousness, followed by a lucid
    interval and then by neurologic deterioration.
    Epidural hematomas are frequently associated
    with linear skull fractures, which cause injury
    to the middle meningeal artery located immediately deep to the overlying fracture. They are
    more common in younger individuals, because
    in younger people, the dura mater is less firmly
    adherent to the inner table of the skull. Todd’s
    phenomenon is a transient focal weakness or
    paralysis that results after a seizure. The particular pattern of weakness is often a clue to the
    site of the seizure focus within the brain.
How well did you know this?
1
Not at all
2
3
4
5
Perfectly
28
Q
  1. Following a sudden impact in an accident, the
    34-year-old race car driver becomes unconscious and is admitted to the hospital. A CT
    scan is performed, and a right space-occupying
    lesion is noted (Fig. 11–1). What is the most
    likely diagnosis?
    (A) Corpus callosum injury
    (B) Pituitary apoplexia
    (C) Acute subdural hematoma
    (D) Acute epidural hematoma
    (E) Chronic subdural hematoma
A
  1. (C) Acute subdural hematomas (Fig. 11–1)
    occur most commonly when violent accelerations or deceleration injuries of the head cause
    tearing of the bridging veins within the subdural potential space. They generally imply a
    much more severe injury to the brain itself than
    in the case of their epidural counterpart. For
    this reason, they are associated with cerebral
    contusions in over 30% of cases.
How well did you know this?
1
Not at all
2
3
4
5
Perfectly
29
Q
  1. A 44-year-old woman was brought to the emergency department after she was involved in a
    high-speed motor vehicle accident. She was
    extracted from the wreckage by paramedics.
    She was intubated at the site and rushed to the
    emergency department. On arrival, her blood
    pressure was 160/80 mm Hg and heart rate
    was 100 bpm, and exam showed evidence of
    decerebrate rigidity. A CT scan of the head
    revealed small punctate hemorrhages in the
    corpus callosum and the midbrain tegmentum,
    but there was no mass effect on adjacent structures. The size of the ventricles was normal.
    This grave clinical presentation and these CT
    findings are most consistent with the diagnosis
    of which of the following?
    (A) DAI
    (B) Cerebral contusion
    (C) Cerebral concussion
    (D) Traumatic subarachnoid hemorrhage
    (SAH)
    (E) Petrous temporal lobe fracture
A
  1. (A) This entity is caused by sharp accelerations
    or decelerations of the head and its contents as
    seen in high-speed motor vehicle accidents.
    During impact, shock waves are generated that
    are able to travel through the semisolid substance
    of the brain. These shock waves penetrate and
    254 11: Neurosurgery
    cause shear and stretch injury to multiple deep
    axonal tracts. DAI represents a severe diffuse
    injury to the entire brain. For this reason, victims present with marked neurological dysfunction. CT scan typically shows no evidence or
    reason to suspect increased ICP; it merely shows
    punctate hemorrhages in many of the tracts that
    are affected.
30
Q
  1. A 43-year-old man presents to the emergency
    department after falling down a flight of stairs
    and landing on his head. He did not lose consciousness. He complains of severe headache,
    marked decreased acuity in hearing in the left
    ear, and a “runny nose” since the fall. On physical examination, he is found to have a leftsided Battle’s sign (an ecchymosis in the area of
    the left mastoid process) and hemotympanum.
    He has a constant dripping of a clear, watery
    fluid through his nose. Findings on his neurologic examination, other than the hearing loss,
    are completely normal. X-ray studies will
    reveal which of the following?
    (A) A fracture of the cribriform plate with a
    CSF leak into the paranasal sinuses
    (B) A skull-base fracture with a mucocele
    (C) A temporal bone fracture with
    paradoxical rhinorrhea
    (D) Occipital bone fracture
    (E) Fracture of the maxillary antrum and
    greater wing of the sphenoid
A
  1. (C) The presence of a Battle’s sign and hemotympanum is highly suggestive of the possibility of a left temporal bone fracture. When
    this occurs, it is common for the dura mater at
    this site to be torn. This leads to leakage of CSF
    into the mastoid air cells and middle ear. CSF
    is subsequently able to reach the nasopharynx
    via the eustachian tube, a phenomenon called
    paradoxical rhinorrhea, which is a serious but
    usually self-limiting condition. Most cases of
    traumatic CSF leaks heal spontaneously within
    approximately 1 week. Patients require close
    in-hospital observation, however, because bacterial meningitis readily occurs in the presence
    of CSF leakage to the outside.
31
Q
  1. A 52-year-old painter injured his lower back 3
    weeks ago when he fell off a ladder. He presents for evaluation of abnormal findings on
    plain radiographs of his lumbar spine. His pain
    has subsided, and he is now asymptomatic.
    Physical examination reveals a dense tuft of
    hair in his lumbosacral region that has been
    present for as long as he can remember. There
    is no tenderness or palpable abnormality in his
    spine. Findings on his neurologic examination
    are unremarkable. The radiographs mentioned
    show absence of the spinous processes and
    laminae at the levels of L5 and S1, with their
    corresponding pedicle displaced and angled
    laterally. What is the diagnosis?
    (A) An L5–S1 spondylolisthesis
    (B) A burst fracture of L5 and S1
    (C) Spina bifida
    (D) Spinal stenosis
    (E) Fracture of the vertebral bodies and
    nucleus pulposus
A
  1. (C) Spina bifida occulta does not cause symptoms and is frequently found incidentally in the
    workup of other conditions. The presence of a
    tuft of hair and the radiographic abnormalities
    described above are consistent with the diagnosis of spina bifida occulta. This is a congenital
    abnormality that results from abnormalities in
    the development of mesodermal elements (sclerotome) which form the dorsal elements of the
    lumbosacral spine. A burst fracture of the spine
    is found after acute excessive axial loading of the
    spine. The features of such a fracture are reduced
    height of the affected vertebral body and displacement of bony fragments centrifugally in
    the axial plane (hence the term burst).
32
Q
  1. In the investigation of chronic back pain, a
    72-year-old man is found on radiologic examination to have congenital spondylolisthesis.
    The pathology is based upon disruption
    between two adjacent vertebra at which site?
    (A) Bodies and disks
    (B) Spinous process
    (C) Transverse process
    (D) Articular process(pars interarticularis)
    (E) Pedicle
A
  1. (D) Spondylolisthesis occurs when there is disruption, most often by a fracture, of the pars
    intra-articularis of the L5 vertebra. The pars is
    the bony element that is found between the
    ascending facets of L5 (that articulate with the
    L4 vertebra) and the descending facets of L5
    (that articulate with S1). The functional result of
    this disruption is that the descending facets are
    “floating” and not able to function in stabilizing the L5–S1 joint. If this becomes progressive, then anterior subluxation of the L5
    vertebral body with respect to that of S1 occurs.
33
Q
  1. A baby is born with a 2.5- × 2.0-cm myelomeningocele in the mid to lower lumbar region.
    Just hours after birth, he is rushed to the operating room (OR) for repair of this defect.
    Approximately 48 hours later, the baby is doing
    well, but it is noted that his head circumference
    has increased by 2 cm. On examination, the
    fontanelle is found to be slightly bulging and
    tense. On neurologic examination, the baby is
    awake but is found to have no spontaneous sensory or motor function below approximately the
    L3 dermatome. An ultrasound of the brain is
    obtained through the open fontanelle. This study
    shows an enlarged ventricular system, consistent with the presence of hydrocephalus. What is
    the related abnormality responsible for the
    hydrocephalus?
    (A) A fourth-ventricle ependymoma
    (B) Stenosis of the aqueduct of Sylvius
    (C) Amelia (failure of limbs to develop)
    (D) Arnold-Chiari malformation
    (E) Nasopharyngeal hamartoma
A
  1. (D) There is a high degree of correlation in the
    occurrence of defects in neural tube closure and
    Arnold-Chiari malformations, and all babies
    born with one should be examined for the other.
    Answers: 24–33 255
    Figure 11–5.
    Epidural hematoma. CT of the head windowed for brain (left) and bone (right)
    shows and epidural hematoma resulting from an underlying occipital skull fracture.
    This injury was caused by a blow to the back of the head. Notice the classic lensshaped hematoma. The brain window also shows a thin left tentorial subdural
    hematoma appearing as a white line running from the midline posteriorly and curving toward the left of the pons. (Reproduced, with permission, from Doherty GM:
    Current Surgical Diagnosis and Treatment, 12th ed. 876. McGraw-Hill, 2006.)
    Development of communicating hydrocephalus
    is a feature of a type-II Arnold-Chiari abnormality. Stenosis of the aqueduct of Sylvius and
    the presence of an ependymoma in the fourth
    ventricles are other reasons for the development of hydrocephalus in children. There is,
    however, no incidental correlation between
    these and defects of neural tube closure.
34
Q
  1. A 4-month-old infant has undergone surgical
    treatment for meningomyeloencephalocele. A
    CT tomogram of head was made immediately
    after birth (see Fig. 11–2). At birth, an operation
    was carried out in the posterior cranial fossa to
    partially replace brain cerebellar contents to
    an intracranial position. In investigations for
    progressive hydrocephalus, it is noted that
    there is herniation of the cerebellar tonsils
    through the foramen magnum, and a diagnosis of Arnold-Chiari syndrome is established.
    This syndrome may also include which of the
    following?
    (A) Fusion of the frontal lobes
    (B) Fusion of the temporal, parietal, and
    occipital lobes
    244 11: Neurosurgery
    (C) Abnormal elongation of the medulla
    and lower cranial nerves
    (D) Partial or complete absence of the
    pituitary gland
    (E) Hypertrophy of cerebral lobes
A
  1. (C) Abnormal elongation of the medulla and
    lower cranial nerves may be evident in ArnoldChiari syndrome. Additional features include
    fusion of the corpora quadrigemina, leading to a
    “beaked” tectum; partial or complete absence of
    the corpus callosum; and microgyria. The corpora quadragemina are relay stations for hearing
    (inferior corpora quadragemina) and the light
    reflex (superior copora quadragemina), and they
    form the posterior surface of the midbrain.
35
Q
  1. During a regular visit to the pediatrician 1 week
    after birth, an infant’s size and head circumference are recorded as being in the seventy-fifth
    percentile. Repeat measurement 1 month later
    still shows the size of the baby at the seventy-fifth
    percentile, but the baby’s head circumference is
    now at the ninty-fifth percentile. The pediatrician
    notices that the baby’s anterior fontanelle is tense
    and that the skull sutures are open. He obtains an
    MRI of the brain with intravenous contrast. This
    study shows the presence of greatly dilated lateral and third ventricles. The aqueduct of Sylvius
    cannot be easily visualized. The fourth ventricle
    is small. There are no lesions within the subarachnoid space or cerebral parenchyma. The
    appearance of the MRI is consistent with which
    of the following?
    (A) Noncommunicating hydrocephalus
    (B) Communicating hydrocephalus
    (C) Normal-pressure hydrocephalus
    (D) Arnold-Chiari malformation with
    herniation of the cerebellum into the
    foramen magnum
    (E) Anencephalus
A
  1. (A) Noncommunicating hydrocephalus is defined as hydrocephalus caused by obstruction
    of CSF flow and obstruction within the ventricular system. In this case, the ventricular system
    is dilated upstream from the obstruction caused
    by stenosis of the aqueduct of Sylvius and collapsed distally. Communicating hydrocephalus
    occurs when the obstruction to CSF flow occurs
    within the subarachnoid space or at the level of
    its resorption into the bloodstream by the arachnoid granulations. In this case, all ventricles are
    dilated proportionately.
36
Q
  1. A 64-year-old woman complains of gait imbalance, headache and deterioration of mental
    status over the past several months. Her vision
    is normal. A CT scan reveals hydrocephalus,
    but the lumbar puncture pressure is unexpectedly low. What does she have?
    (A) Meningitis
    (B) Normal-pressure hydrocephalus
    (C) Sigmoid sinus thrombosis
    (D) Echinococcus
    (E) Glioblastoma multiforme
A
  1. (B) Normal-pressure hydrocephalus is a condition seen in the elderly in which there is symmetrical enlargement of the entire ventricular
    system. When patients with this condition are
    studied by lumbar puncture, it is found that
    despite ventriculomegaly, the ICP is abnormally low. This syndrome presents with a characteristic triad of symptoms—dementia, ataxia,
    and urinary incontinence.
37
Q
  1. A 23-year-old woman complains of progressive
    loss of vision and papilledema. Investigations
    show normal findings on CT scan. A lumbar
    puncture shows marked elevation of pressure.
    What is the most likely diagnosis?
    (A) Pseudotumor cerebri
    (B) Corpus cavernous thrombosis
    (C) Cavernous sinus thrombosis
    (D) Retinoblastoma
    (E) Chordoma
A
  1. (A) Pseudotumor cerebri is a condition that
    most commonly occurs in young adults, particularly in females. In this condition, ICP as
    measured by a lumbar puncture is elevated,
    while the size of the cerebral ventricles on imaging studies is small or normal. It is a generally
    progressive condition that causes headache and
    damage to the optic nerve, sometimes leading
    to loss of peripheral vision and blindness.
38
Q
  1. During her eighth month of pregnancy, a
    29-year-old woman is noted to have hydramnios. Further testing shows anencephalus. In
    this case hydramnios is caused by which of the
    following?
    (A) Impairment of the fetus’s swallowing
    mechanism
    (B) Tumor of the fetus’s brain
    (C) A secretory peptide from the placenta
    (D) Excess antidiuretic hormone (ADH)
    from the fetus
    (E) Renal agenesis
A
  1. (A) This abnormality is relatively common and
    occurs in 1 of 1000 pregnancies. It occurs four
    times more commonly in whites than blacks and
    four times more commonly in female fetuses
    than in male fetuses. The abnormality can be
    identified on an x-ray, because the vault of the
    skull is absent. Anencephalus is caused by failure
    of the cephalic part of the neural tube to close off.
39
Q
  1. A 28-year-old man presents with a history of
    chronic headache. The headache is intermittent, severe, poorly localized, and most often
    present when he arises in the morning. He suffered a severe blow to the head and sustained
    a skull fracture at the age of 15. Findings on his
    physical and neurologic examinations are
    within normal limits. An MRI of the brain with
    gadolinium reveals the presence of a large,
    nonenhancing extra-axial cyst in the region of
    the right temporal tip. This most likely represents which of the following?
    (A) An arachnoid cyst
    (B) A cystic astrocytoma
    (C) Rathke’s cleft cyst
    (D) A Dandy-Walker cyst (failure of proper
    formation of the foramina of Lushka
    and Magendie)
    (E) Polycystic disease
A
  1. (A) This cystic structure is an arachnoid cyst.
    These are CSF-filled cysts that occur when
    leaves of arachnoidal tissue fuse, trapping CSF
    within them. These cysts slowly grow over
    time, sometimes attaining very large size. They
    cause symptoms by virtue of their large size, as
    they are able to compress adjacent structures.
    Patients with these cysts most commonly
    present with a history of chronic headache.
    Neurologic symptoms or deficits are unusual.
    Patients with arachnoid cysts frequently give a
    history of severe blows to the head and skull
    fractures, perhaps implying head trauma as a
    causative agent. The most common locations of
    arachnoid cysts are the middle cranial fossa,
    the cerebellopontine angle, and the suprasellar
    area. Dandy-Walker cysts are the result of an
    intrauterine developmental abnormality in
    which there is failure of proper formation of the
    foramina of Lushka and Magendie. As a consequence, the main egress of CSF out of the
    ventricular system is obstructed, leading to
    hydrocephalus and a massively enlarged, cystlike fourth ventricle.
40
Q
  1. A 15-year-old boy complains of right-sided
    weakness and gait impairment. ACT scan shows
    a large, nonenhancing cyst in the posterior
    cranial fossa, with an enhancing tumor nodule in
    the left cerebellum. What is the most likely
    diagnosis?
    (A) An arachnoid cyst
    (B) A cystic astrocytoma
    (C) Rathke’s cleft cyst
    (D) Glioblastoma multiforme
    (E) A large sebaceous cyst
A
  1. (B) Cystic astrocytomas are neoplasms of the
    CNS. They usually consist of a large, nonenhancing cyst on the wall of which is an enhancing tumor nodule. They are most commonly
    found within the substance of the cerebellar
    hemispheres of children and young adults. A
    Rathke’s cleft cyst is a remnant of the embryologic Rathke’s pouch. These are found within
    the sella turcica.
41
Q
  1. A 56-year-old woman presents with a history of
    several months of pain involving both hands.
    She describes the pain as electric and severe. It
    is localized to the palmar aspect of the first three
    digits of each hand and associated with numbness. The pain is particularly severe in the
    morning when she wakes up. She reports no
    weakness of the hands, but she says that sometimes objects fall off her hand because she
    cannot feel them. Physical examination reveals
    atrophy and weakness in the muscles of the
    thenar eminence bilaterally. She also has numbness in the distribution of the median nerve
    within the hands. Phalen test is positive. Which
    is the best test to confirm the clinical diagnosis?
    (A) An MRI of the hand to visualize an
    enlarged carpal ligament
    (B) An EMG and nerve-conduction study
    (C) MRI of the cervical spine to rule out
    radiculopathy
    (D) An x-ray of the hand
    (E) Physical examination
A
  1. (B) CTS is a condition in which the median
    nerve is compressed at the level of the wrist by
    a thickened carpal flexor retinaculum. This
    256 11: Neurosurgery
    leads to numbness and painful paresthesia
    along the median nerve distribution within the
    hand. It also causes weakness and atrophy of
    the thenar muscles within the hand, innervated
    by the superficial recurrent branch of the
    median nerve. Once there is clinical suspicion,
    the best diagnostic test to confirm the presence
    of CTS is a nerve-conduction study. This study
    often shows a block or delay in conduction of
    the median nerve at the level of the carpal
    tunnel. Conduction within all branches of the
    ulnar nerve should be normal. This test is often
    also useful in distinguishing between CTS and
    the possibility of a C6 radiculopathy.
42
Q

A 28-year-old police officer is brought to the emergency room (ER) by ambulance following a gunshot to
the head. Emergency medical services (EMS) reports
that he was found unresponsive at the site of the shooting and was immersed in a pool of blood. There were
no witnesses. On arrival to the emergency department, he is noted to have a bullet entry wound on the
right frontal region without any exit wound. His blood
pressure is 80/35 mm Hg, pulse rate 150 bpm, and on
examination, he does not open his eyes or follow commands. He is unresponsive to deep painful stimuli
such as testing by sternal rub. His pupils are dilated
approximately 4 mm bilaterally, but sluggishly reactive. He is aggressively resuscitated with colloid and blood products. The blood pressure is now 140/75
mm Hg. There is improvement in his neurologic examination—1 hour after admission, he withdraws his
limbs from painful stimuli. A CT scan shows a smallskull defect in the right frontal region, representing the
bullet entry site. The bullet is lodged within the cerebral parenchyma, approximately 2 cm from the surface
of the brain, and there is a trail of bone fragments
along the bullet path. The bullet has not crossed the
midline. There is a 2 × 2 × 2.5-cm hematoma within the
substance of the right frontal lobe with surrounding
edema and subfalcian herniation.

  1. Which item is least likely to be useful as a prognostic marker for subsequent recovery?
    (A) Neurologic examination upon
    presentation and early response
    (B) The fact that the bullet did not cross the
    midline
    (C) The presence of an intracerebral
    hematoma
    (D) The presence of edema with subfalcian
    herniation
    (E) Bullet crosses the midcoronal plane
A
  1. (A) The best prognostic indicator of survival
    and outcome in patients with missile wounds to
    the brain is the mental status and level of
    responsiveness after proper resuscitation. His
    initial poor neurologic grade can be attributed
    to cerebral injury itself or to cerebral hypoperfusion in a patient with clear hemodynamic
    shock. Initial presentation is, thus, of little value
    in judging the prognosis for these types of
    injuries. Other prognostic factors that have been
    identified as important in predicting the outcome of gunshot wounds to the head include:
    (a) Path of the bullet. A missile that crosses
    the midline or the midcoronal plane is
    associated with a much worse outcome
    than one that stays unilaterally.
    (b) The presence of an intracranial
    hematoma of greater than 2 × 2 × 2 cm is
    ironically a positive prognosticator,
    because it represents a mass lesion that
    can be causing intracranial hypertension
    and can be more readily evacuated via a
    craniotomy.
43
Q

A 28-year-old police officer is brought to the emergency room (ER) by ambulance following a gunshot to
the head. Emergency medical services (EMS) reports
that he was found unresponsive at the site of the shooting and was immersed in a pool of blood. There were
no witnesses. On arrival to the emergency department, he is noted to have a bullet entry wound on the
right frontal region without any exit wound. His blood
pressure is 80/35 mm Hg, pulse rate 150 bpm, and on
examination, he does not open his eyes or follow commands. He is unresponsive to deep painful stimuli
such as testing by sternal rub. His pupils are dilated
approximately 4 mm bilaterally, but sluggishly reactive. He is aggressively resuscitated with colloid and blood products. The blood pressure is now 140/75
mm Hg. There is improvement in his neurologic examination—1 hour after admission, he withdraws his
limbs from painful stimuli. A CT scan shows a smallskull defect in the right frontal region, representing the
bullet entry site. The bullet is lodged within the cerebral parenchyma, approximately 2 cm from the surface
of the brain, and there is a trail of bone fragments
along the bullet path. The bullet has not crossed the
midline. There is a 2 × 2 × 2.5-cm hematoma within the
substance of the right frontal lobe with surrounding
edema and subfalcian herniation.

43. What is the next step in management?
(A) Administration of mannitol (1 g/kg)
through a rapid IV infusion followed by
the placement of an intracranial pressure
monitor
(B) Administration of mannitol (1 g/kg)
through a rapid IV infusion followed by
urgent craniotomy
(C) Administration of mannitol (1 g/kg)
through a rapid IV infusion followed by
the placement of burr holes for emergent
decompression of raised intracranial
pressure
(D) No treatment should be administered,
because the patient’s prognosis is poor,
and he is unlikely to survive
(E) Steroids and antibiotics alone
A
  1. (B) A markedly diminished level of consciousness coupled by a CT scan that shows a
    hematoma, edema, and subfalcian herniation
    indicate that the patient is suffering from
    intracranial hypertension. Hyperventilation and
    mannitol are quick and effective ways to reduce
    intracranial pressure temporarily. However,
    these measures are only temporary, and the
    patient needs urgent decompression by craniotomy. Placement of burr holes in the ER is of no
    value in the management of these injuries.
    Placement of an ICP monitor may be helpful for
    the postoperative period, but is likely to be of
    limited help without prior craniotomy.
44
Q

A 28-year-old police officer is brought to the emergency room (ER) by ambulance following a gunshot to
the head. Emergency medical services (EMS) reports
that he was found unresponsive at the site of the shooting and was immersed in a pool of blood. There were
no witnesses. On arrival to the emergency department, he is noted to have a bullet entry wound on the
right frontal region without any exit wound. His blood
pressure is 80/35 mm Hg, pulse rate 150 bpm, and on
examination, he does not open his eyes or follow commands. He is unresponsive to deep painful stimuli
such as testing by sternal rub. His pupils are dilated
approximately 4 mm bilaterally, but sluggishly reactive. He is aggressively resuscitated with colloid and blood products. The blood pressure is now 140/75
mm Hg. There is improvement in his neurologic examination—1 hour after admission, he withdraws his
limbs from painful stimuli. A CT scan shows a smallskull defect in the right frontal region, representing the
bullet entry site. The bullet is lodged within the cerebral parenchyma, approximately 2 cm from the surface
of the brain, and there is a trail of bone fragments
along the bullet path. The bullet has not crossed the
midline. There is a 2 × 2 × 2.5-cm hematoma within the
substance of the right frontal lobe with surrounding
edema and subfalcian herniation.

  1. Intraoperative management of this patient
    should be avoidance of which of the following?
    (A) Placement of an intracranial pressure
    monitor
    (B) Performance of a wide craniotomy for
    evacuation of the intraparenchymal
    hematoma
    (C) Extensive debridement of all bullet and
    bone fragments
    (D) Reconstruction of the cranial defect
    caused by the bullet
    (E) Removal of necrotic brain material
A
  1. (C) The ideal intraoperative management of
    this patient would begin by performance of a
    wide craniotomy through which the intracerebral hematoma can be evacuated. Necrotic
    brain tissue if left alone is likely to worsen the
    occurrence of cerebral edema postoperatively,
    and for that reason, every measure should be
    taken to debride it as thoroughly as possible.
    Easily accessible bone and bullet fragments can
    also be removed. Bone and bullet fragments
    that are deeply located and difficult to locate
    should be left intact. Persistence in their
    removal often leads to a greater risk of brain
    injury by intraoperative manipulation and dissection. If problems with raised intracranial
    pressure are expected, placement of a suitable
    ICP monitoring device is highly recommended
    as part of the surgical procedure.
45
Q

A 54-year-old-man comes to the emergency department complaining of a severe headache for several
hours. He describes this headache as the worst of his
life. It started suddenly “like a firecracker had gone
off” inside his head. He has had no loss of consciousness but has had several episodes of vomiting.
General physical examination reveals a patient who
is in severe distress due to the headache. His blood
pressure is 180/70 mm Hg, and his pulse racing at
120 bpm. He is afebrile. He has photophobia and
gross neck rigidity. Neurologically, he is fully alert
and oriented. He has a normal motor and sensory
examination. His left pupil is 2 mm and briskly reactive to light; his right is 4.5 mm and fixed to both
light and accommodation.

45. What is the most likely diagnosis?
(A) Acute bacterial meningitis
(B) Incipient uncal herniation due to an
expanding lesion in the right temporal
lobe
(C) Acute SAH from an anterior
communicating artery aneurysm
(D) Acute SAH from a right posterior
communicating aneurysm
(E) Cavernous sinus thrombosis
Questions: 37–45 247
A
  1. (D) This is the classic history for acute SAH—the
    acute onset of a massive headache. The acuity
    should suggest nothing other than a vascular
    phenomenon. Furthermore, the presence of a
    right occulomotor nerve palsy strongly suggests
    bleeding from an aneurysm of the right posterior communicating artery. Anatomically, most
    posterior communicating aneurysms point their
    domes laterally and inferiorly, in the direction
    toward the occulomotor nerve. In general, when
    the dome of the aneurysm ruptures, the jet of
    blood injures the adjacent nerve. In this situation, the lesion results in complete occulomotor
    nerve palsy with a fixed dilated pupil. It is a
    neurosurgic dogma that complete occulomotor
    palsy should be regarded as a ruptured posterior
    communicating artery aneurysm until proved
    otherwise. Acute bacterial meningitis also presents with headache and meningism. The onset
    of the symptoms is, however, much more gradual, and high fever is usually present.
46
Q

A 54-year-old-man comes to the emergency department complaining of a severe headache for several
hours. He describes this headache as the worst of his
life. It started suddenly “like a firecracker had gone
off” inside his head. He has had no loss of consciousness but has had several episodes of vomiting.
General physical examination reveals a patient who
is in severe distress due to the headache. His blood
pressure is 180/70 mm Hg, and his pulse racing at
120 bpm. He is afebrile. He has photophobia and
gross neck rigidity. Neurologically, he is fully alert
and oriented. He has a normal motor and sensory
examination. His left pupil is 2 mm and briskly reactive to light; his right is 4.5 mm and fixed to both
light and accommodation.

46. What is the most appropriate test to establish
the diagnosis?
(A) MRI of the brain with and without
gadolinium
(B) CT scan of the brain without contrast
(C) A lumbar puncture
(D) An electroencephalogram
(E) Optometry
A
  1. (B) The best test in the diagnosis of an acute
    SAH is a nonenhanced CT of the brain. In this
    study, subarachnoid blood can easily be seen as
    a hyperdense substance filling the otherwise
    Answers: 34–46 257
    hypodense cisterns of the subarachnoid space.
    Its sensitivity is greater 95%, but sensitivity
    falls to 50% by 1 week after the hemorrhage.
    Lumbar puncture can also be used to diagnose
    SAH, but it is an invasive procedure that
    should be reserved for cases in which the suspicion of such hemorrhage remains following
    a negative CT scan. MRI (with or without
    gadolinium), despite its exquisite sensitivity
    for the diagnosis of intracerebral lesions, is
    notoriously poor in its ability to detect acute
    blood within the subarachnoid space. EEG is of
    no value for the diagnosis of an acute SAH.
47
Q
  1. A 43-year-old man is treated with pyridostigmine for facial, ocular, and pharyngeal weakness due to myasthenia gravis. Which statement
    is true of pyridostigmine?
    (A) It is unrelated to neostigmine.
    (B) It has far more side effects than
    neostigmine.
    (C) Pyridostigmine and neostigmine reverse
    depolarizing neuromuscular blockade.
    (D) It causes greater muscarinic effect than
    neostigmine.
    (E) It is an anticholinesterase agent.
A
  1. (E) Neostigmine and pyridostigmine are both
    anticholinesterase agents and can be used in
    the reversal of nondepolarizing muscle relaxants. Pyridostigmine causes less muscarinic
    effect than does neostigmine. The effect of pyridostigmine is more prolonged and produces
    fewer secretions and less severe bradycardia.
48
Q
  1. During anesthesia using a narcotic, thiopental,
    and N2
    O, the respiratory response to a rising
    end-respiratory CO2 tension is which of the following?
    (A) Depressed only by the narcotic
    (B) Depressed only by thiopental
    (C) Depressed progressively by the addition
    of each agent
    (D) Depressed by the narcotic and thiopental,
    then elevated by N2
    O
    (E) Unchanged from control response
A
  1. (C) Both narcotics and thiopental depress respiration, and the addition of N2
    O further augments this depressant action. Thus, the response
    to hypercapnea is diminished.
49
Q
  1. A plastic surgeon is performing a minor procedure on the face of an 18-year-old woman.
    She has a seizure that is attributed to the local
    anesthetic agent. Convulsion following an
    overdose of local anesthesia is best treated by
    which of the following?
    (A) Droperidol
    (B) Hydroxyzine (Vistaril)
    (C) Diazepam (Valium)
    (D) Fentanyl ketamine
A
  1. (C) Diazepam is a benzodiazepine derivative
    that seems to have a calming effect on part of the
    limbic system, thalamus, and hypothalamus. It
    should be injected slowly (<1 mg/min) into a
    larger vein to avoid phlebitis and local irritation.
50
Q
  1. A 17-year-old male presents with 3-month
    history of headache, weight gain, decreased
    concentration, polyuria, and polydypsia. His
    headaches are mostly in morning and involves
    the frontal region. On examination he was found
    to have bitemporal visual field defect and no
    facial hair. MRI scan revealed a suprasellar partially calcified cystic lesion with displacement of
    optic chiasm. The most likely pathology is:
    (A) Giant aneurysm of carotid artery
    (B) Pituitary macroadenoma
    (C) Glioblastoma multiforme
    (D) Craniopharyngioma
    (E) Testicular metastasis
A
  1. (D) Weight gain, DI, decreased memory, and
    visual field defect are consistent with a suprasellar hypothalamic lesion. Calcified cystic lesions
    in this location, particularly in adolescents, are
    characteristic of craniopharyngiomas. The rate
    of calcification in childhood is about 85% and in
    adult is about 40%. Craniopharyngiomas are
    pathologically benign but due to their location
    and their firm attachment to critical structures
    surrounding them can result in severe neurofunctional impairment. The cyst wall is lined
    with squamous epithelium and the fluid contains cholesterol crystals.
51
Q
51. He underwent a craniotomy for resection of
his lesion. Twelve hours postoperatively, he
developed diuresis of over 500 mL/h. The
diagnosis of (DI) was entertained. What laboratory findings are most consistent with the
clinical impression?
(A) Urine specific gravity of over 1010
(B) Serum sodium of less than 135
(C) Decreased both serum and urine
osmololity
(D) Increased serum osmololity and
decreased urine osmololity
(E) Increased both serum and urine
osmololity
A
  1. (D) DI is commonly precipitated by low level of
    ADH secretion. The clinical presentation does
    not manifest until over 85% of ADH secretory
    capacity is damaged. Rarely it is nephrogenic
    and is caused by lack of renal response to ADH
    hormone. Nephrogenic form is often produced
    by toxic effect of certain drugs or familial Xlinked recessive genetic disorder. DI is defined
    by increased diluted urinary output, constant
    thirst for usually cold water, and high serum
    osmololity. If it is not properly treated it can
    lead to extreme dehydration and electrolyte
    imbalance. The best test is water deprivation
    for 4 hours while monitoring the urine output
    and urine and serum osmololity. If urine
    osmololity remains flat or changes less than 30
    mOsm and the serum osmololity approaches
    300 mOsm/L, the diagnosis of DI is confirmed.
    At this point the patient should be given 5 U of
    an exogenous pitressin subcutaneously. By
    comparing the urine osmololity after Pitressin
    to the initial values, the extent of the DI can be
    determined. The increase in urine osmololity
    by more than 67% is indicative of severe ADH
    deficiency. The increased levels of 6–67% is suggestive of partial deficiency.
52
Q
  1. A 55-year-old female presents with 3-years history of severe lancinating pain extending from
    left ear to her maxillary area. Pain is triggered
    by chewing and brushing teeth. She was
    treated by otolaryngologist for sinus infection
    a year ago and undergone multiple dental
    work and teeth extraction with transient or no
    improvement. The most likely diagnosis is:
    (A) Maxillary sinusitis
    (B) Trigeminal neurolgia
    (C) Maxillary osteomyelitis
    (D) Gradenigo’s syndrome
    (E) Otitis media
A
  1. (B) Trigeminal neuralgia or Tic Douloureux is
    clinically characterized by paroxysmal lancinating pain in the distribution of one or two
    division of trigeminal nerve. There is commonly
    no sensory or motor impairment on examination. The pain is triggered by certain mild stimuli such as touching, chewing, brushing teeth,
    and cold breeze. If any objective neurological
    findings are detected, other pathologies causing
    compression of the nerve at its exit zone from
    the brainstem must be suspected. Such pathologies as AVM, tumors, and aneurysm must be
    ruled out by MRI. This disease is assumed to be
    caused by a loop of a vessel, often superior
    cerebellar artery or posterior inferior cerebellar
    artery (PICA), compressing the trigeminal
    nerve as it emerges from brainstem.
53
Q

A 55-year-old female presents with 3-years history of severe lancinating pain extending from
left ear to her maxillary area. Pain is triggered
by chewing and brushing teeth. She was
treated by otolaryngologist for sinus infection
a year ago and undergone multiple dental
work and teeth extraction with transient or no
improvement.

  1. Which one of the following medications is not
    indicated in treatment of this condition?
    (A) Carbamezapin
    (B) Cefatin
    (C) Phenytoin
    (D) Gabapentin
    (E) Baclofen
A
  1. (B) Most antiepileptic medications are effective to
    control the pain in this condition. The drug of
    choice is Carbamazepin which is effective in twothirds of the patients. Phenytoin is an intravenous
    option in those who cannot take oral medication
    due to severe pain. Baclofen is an effective medication in conjunction with Carbamazepin.
    Gabapentin is another antiepileptic medication
    258 11: Neurosurgery
    which is useful in mild forms of the TN or in
    association with other medications. Other medications that are being used for this condition
    include Amitriptyline (Elavil) and Clonazepam
    (Klonopin).
54
Q
  1. A 45-year-old woman was brought to emergency
    department for sudden onset of severe headache
    associated with photophobia, nausea, and transient loss of consciousness. On examination, she
    is awake and alert with normal cranial nerve
    function. She also exhibits normal muscle
    strength and sensation.Her past medical history
    is significant for sickle cell disease (SCD) and
    hypertension. CT scan confirms the diagnosis of
    SAH without any intraparenchymal abnormality.
    What is the least likely cause of SAH?
    (A) Aneurysmal bleed
    (B) Sickle cell angiopathy
    (C) Arteriovenous malformation (AVM)
    (D) Hemorrhagic meningioma
    (E) Blood dyscrasia
A
  1. (D) The most common cause of the spontaneous
    SAH is berry aneurysm and AVM. Aneurysms
    are often located on the circle of Willis in the
    subarachnoid space hence bleeding occurs in
    this space. Massive hemorrhages can break in
    the parenchyma and produce intraparenchymal
    clot. AVMs usually bleed in the parenchyma but
    frequently is superficial and present with associated SAH. Sickle cell angiopathy can also present as SAH from subpial vessels. In a patient
    with history of SCD, angiopathy as the cause of
    SAH should be considered. On other hand,
    bleeding is extremely rare in meningiomas and
    when it occurs it is often intratumoral or intraparenchymal and do not appear as a SAH.
55
Q

A 45-year-old woman was brought to emergency
department for sudden onset of severe headache
associated with photophobia, nausea, and transient loss of consciousness. On examination, she
is awake and alert with normal cranial nerve
function. She also exhibits normal muscle
strength and sensation.Her past medical history
is significant for sickle cell disease (SCD) and
hypertension. CT scan confirms the diagnosis of
SAH without any intraparenchymal abnormality.

55. What is the most definitive diagnostic test in
this condition?
(A) CT angiography
(B) Magnetic resonance angiography (MRA)
(C) Cerebral angiogram
(D) MR spectroscopy
(E) Positron emission tomography (PET)
scan
A
  1. (C) Four vessel angiography remains a gold standard vascular study for detecting aneurysms
    and AVMs. It provides the detail and definition
    that are required for surgical intervention. MRA
    and CTA are often very helpful to detect and
    provide some detail. The newer high speed CT
    scans with angiography software can produce
    the definition close to standard angiography. PET
    scan and spectroscopy are primarily used to
    determine the metabolic activities of the brain
    and are incapable of detecting vascular lesions.
56
Q

A 45-year-old woman was brought to emergency
department for sudden onset of severe headache
associated with photophobia, nausea, and transient loss of consciousness. On examination, she
is awake and alert with normal cranial nerve
function. She also exhibits normal muscle
strength and sensation.Her past medical history
is significant for sickle cell disease (SCD) and
hypertension. CT scan confirms the diagnosis of
SAH without any intraparenchymal abnormality.

56. What is the most likely complication of angiography in this patient?
(A) Cerebral stroke
(B) Aneurysmal rupture
(C) Increased intracranial pressure
(D) Vascular wall damage
(E) Sickle cell crisis
A
  1. (E) The rate of complications in cerebral
    angiography is relatively low and in experienced hands is less than 1%. Among the potential complications, cerebral ischemic or
    hemorrhagic stroke, vascular dissection, and
    aneurysmal rupture are frequently reported.
    In patients with SCD administration of the
    high-osmolar contrast often precipitate sickle
    cell crisis. Therefore, these patients should be
    pretreated with exchange transfusion and
    maintained in a well hydrated state. Steroids
    are also used prior to angiography to reduce
    the postangiography complications.
57
Q
  1. A 69-year-old well-controlled, hypertensive man
    was seen in ER with 3-month history of mild
    headache and sudden onset of hemiparesis. On
    examination, he exhibit mild dysphasia and
    lethargy. His cognitive function testing indicates
    moderate diminution of his recent memory and
    executive function. His hemiparesis is more
    dense in arm and leg and is mild in his face.CT
    scan without contrast demonstrates a 3-cm irregular hemorrhage surrounded by marked edema
    and mass effect in frontal-temporal region. The
    most likely cause of bleed is?
    (A) Amyloid angiopathy
    (B) Hypertensive hemorrhage
    (C) Hemorrhagic neoplasm
    (D) Arterial-venous malformation
    (E) Coagulopathy
A
  1. (C) History of recent onset of headache in adult
    should always raise the suspicion of neoplasm.
    Amyloid angiopathy and hypertension are frequent cause of the intracerebral hemorrhage in
    elderly, but the critical difference with neoplastic bleed is the presence of edema and mass
    effect which is disproportionate to the size of
    the hemorrhage. The presence of edema suggests a preexisting lesion with recent bleed.
    Although intraparenchymal hemorrhage can
    produce edema but it takes many hours to
    develop. Therefore, the presence of edema in
    early hours after the ictus is indicative of
    underlying pathology. AVMs and coagulopathies also can cause intraparenchymal
    bleed without surrounding edema which is the
    hallmark of an underlying lesion.
58
Q

A 69-year-old well-controlled, hypertensive man
was seen in ER with 3-month history of mild
headache and sudden onset of hemiparesis. On
examination, he exhibit mild dysphasia and
lethargy. His cognitive function testing indicates
moderate diminution of his recent memory and
executive function. His hemiparesis is more
dense in arm and leg and is mild in his face.CT
scan without contrast demonstrates a 3-cm irregular hemorrhage surrounded by marked edema
and mass effect in frontal-temporal region.

58. What is the next diagnostic test that should be
ordered?
(A) EEG
(B) Cerebral angiography
(C) MRI with contrast
(D) Spinal tap to determine the ICP
(E) Transcranial Doppler
A
  1. (C) MRI with contrast is the study of choice to
    determine the presence of underlying pathology. The pattern of enhancement can identify
    such lesions as primary brain tumor or hemorrhage in a metastatic lesion. Although lung and
    breast are the most common neoplasm that
    metastasis to brain, the incidence of bleeding is
    more common in melanomas and lymphomas.
    AVMs also have special MRI features that make
    the diagnosis possible. The vascular details
    nonetheless require angiographic studies. The
    vascular study is ineffective in determining the
    presence of neoplastic lesions unless an AVM or
    aneurysms are suspected. Transcranial Doppler
    is used to determine the circulation velocity in
    the intracranial vessels particularly in middle
    cerebral artery. This test is used to monitor the
    vasospasm occurring after aneurysmal SAH and
    has no role in identifying any other pathology.
59
Q

Electromyography (EMG) and nerve conduction studies (NCS) are useful for assessing the function of

A. Peripheral nerves
B. Bilateralcarotidarteries
C. Intracranialliquids
D. Spinalnerves

A

Answer:A
Electromyography and nerve conduction studies (EMG/ NCS) are use ul or assessing the unction o peripheral nerves. EMG records muscle activity in response to a proxi- mal stimulation o the motor nerve. NCS record the velocity and amplitude o the nerve action potential. EMG/NCS typi- cally is per ormed approximately 3 to 4 weeks a ter an acute injury, as nerves distal to the injury continue to transmit elec- tricalimpulsesnormallyuntildegenerationo thedistalnerve progresses. (See Schwartz 10th ed., p. 1712.)

60
Q

T e lesion that can cause mass ef ect and rapidly kill the patient is
A. In erior ossa lesions
B. Posterior ossalesions C. Progressiveobtundation D. Bradycardiallesions

A

Answer:B
heposterior ossa(brainstemandcerebellum)requiresspe- cial consideration because the volume o the posterior ossa within the cranial vault is small. Posterior ossa lesions such as tumors, hemorrhage, or stroke can cause mass e ect that can rapidly kill the patient in two ways. Occlusion o the ourth ventricle can lead to acute obstructive hydrocephalus, raised intracranial pressure (ICP), herniation, and eventually death.
his mass e ect can also lead directly to brain stem com- pression (Fig. 42-1). Symptoms o brain stem compression include hypertension, agitation, and progressive obtundation,
ollowed rapidly by brain death. A patient exhibiting any o these symptoms needs an emergent neurosurgical evalua- tion orpossibleventriculostomyorsuboccipitalcraniectomy (removal o the bone covering the cerebellum). his situation is especially critical, as expeditious decompression can lead to signi icant unctional recovery. (See Schwartz 10th ed., Figure 42-5, pp. 1714–1715.)

61
Q
  1. According to the Glasgow Coma Scale (GCS), a patient withaheadinjuryscoreo 5isclassi edas
    A. Mild
    B. Moderate C. Severe
    D. Seizure
A

Answer:D
raumatic brain injury ( BI) can be classi ied as mild, moder- ate, or severe. For patients with a history o head trauma, clas- si ication is as ollows: severe head injury i the GCS score is 3 to 8, moderate head injury i the Glasgow Coma Scale (GCS)scoreis9to12,andmildheadinjuryi theGCSscoreis 13 to 15. Many patients present to emergency rooms and trauma bays with a history o BI. A triage system must be used to maximize resource utilization while minimizing the chance o missing occult or progressing injuries. (See Schwartz 10th ed., p. 1718.)

62
Q
  1. In regard to the halo test, a positive indicator
    spinal uid (CFS) when tinged with blood will show the
    ollowing when dropped on an absorbent tissue
    A. A single ring with a darker center spot containing
    bloodcomponentssurroundedbyalighthaloo CFS. B. A double ring with a darker center spot containing
    bloodcomponentssurroundedbyalighthaloo CFS.
    C. A single ring with a lighter center spot containing CFS
    surroundedbyadarkerhaloo bloodcomponents.
    D. A double ring with a lighter center spot containing CFS
    surroundedbyadarkerhaloo bloodcomponents
A

Answer:B
Copious clear drainage rom the nose or ear makes the diag- nosis o cerebrospinal luid (CSF) leakage obvious. O ten, however, the drainage may be discolored with blood or small in volume i some drains into the throat. he halo test can help di erentiate. Allow a drop o the luid to all on an absorbent sur ace such as a acial tissue. I blood is mixed with CSF, the drop will orm a double ring, with a darker center spot con- taining blood components surrounded by a light halo o CSF. I this test is indeterminate, the luid can be sent or beta-2 trans errin testing, a carbohydrate- ree iso orm o trans-
errin exclusively ound in the CSF. (See Schwartz 10th ed., pp. 1716–1717.)

63
Q
  1. Neurapraxiaisde nedas
    A.T e disruption o axons and myelin.
    B.T e disruption o axons and endoneurial tubes.
    C.T e temporary ailure o nerve unction without physi-
    cal axonal disruption.
    D. T e temporary ailure o nerve unction with physical
    axonal disruption.
A

Answer:C
Neurapraxia is de ined as the temporary ailure o nerve unc- tion without physical axonal disruption. Axon degeneration does not occur. Return o normal axonal unction occurs over hours to months, o ten in the 2- to 4-week range. (See Schwartz 10th ed., p. 1026.)

64
Q
  1. Apatientwhowithdraws rompain,ismumblinginappro- priate words, and opens his eyes to pain has a GCSscore o
    A. 3 B. 6 C. 9 D. 12
A

Answer:C

65
Q
  1. T emostcommonmalignanttumoro thebrainis A. Ependymoma
    B. Astrocytoma
    C. Ganglioglioma
    D. eratoma
A

Answer:B
Astrocytoma is the most common primary central nervous sys- tem (CNS) neoplasm. he term glioma o ten is used to re er to astrocytomas speci ically, excluding other glial tumors. Astro- cytomas are graded rom I to IV. Grades I and II are re erred to as low-grade astrocytoma, grade III as anaplastic astrocytoma, and grade IV as glioblastoma multiforme (GBM). Prognosis var- ies signi icantly between grades I/II, III, and IV, but not between I and II. Median survival is 8 years a ter diagnosis with a low- grade tumor, 2 to 3 years with an anaplastic astrocytoma, and roughly 1 year with a GBM. GBMs account or almost two- thirdso allastrocytomas,anaplasticastrocytomasaccount or two-thirdso therest,andlow-gradeastrocytomastheremain- der.Figure42-2demonstratesthetypicalappearanceo aGBM. (See Schwartz 10th ed., Figure 42-20, pp. 1733–1734.)

66
Q
  1. A 25-year-old man is seen in the emergency department af er he struck his head against the windshield in an auto- mobile accident. He opens his eyes and withdraws his arm during pain ul stimulation o his hand. He responds verbally to questions with inappropriate words. His GCSscore is
    A. 6 B. 9 C. 12 D. 15
A

Answer:B
heinitialassessmento thetraumapatientincludesthepri- mary survey, resuscitation, secondary survey, and de initive care. Neurosurgical evaluation begins during the primary sur- vey with the determination o the GCS score (usually re erred to simply as the GCS) or the patient. he GCS is determined by adding the scores o the best responses o the patient in each o three categories. he motor score ranges rom 1 to 6, verbal rom 1 to 5, and eyes rom 1 to 4. he GCSthere ore ranges rom 3 to 15, as detailed in able 42-1. racheal intu- bation or severe acial or eye swelling can impede verbal and eye responses. In these circumstances, the patient is given the score o 1 with a modi ier, such as verbal “1 ” where = tube. (See Schwartz 10th ed., p. 1715.)

67
Q
  1. T e most common level o cervical radiculopathy rom cervical disc herniation is
    A. C4–C5 B. C5–C6 C. C6–C7 D. C7– 1
A

Answer:C
he cervical nerve roots exit the central canal above the ped- icle o the same-numbered vertebra and at the level o the higher adjacent intervertebral disc. For example, the C6 nerve root passes above the C6 pedicle at the level o the C5–C6 discs. he cervical nerve roots may be compressed acutely by disc herniation, or chronically by hypertrophic degenerative changes o the discs, acets, and ligaments. able 42-2 sum- marizesthee ectso variousdischerniations.Mostpatients with acute disc herniations. Most patients with acute disc herniations will improve without surgery, nonsteroidal anti- in lammatory drugs (NSAIDs), or cervical traction may help alleviate symptoms. Patients whose symptoms do not resolve
or who have signi icant weakness should undergo decompres- sive surgery. he two main options or nerve root decompres- sionareanteriorcervicaldiscectomyand usion(ACDF)and posterior cervical oraminotomy (keyhole oraminotomy). ACDFallowsmoredirectaccesstoandremovalo thepathol- ogy (anterior to the nerve root). However, the procedure requires usion because discectomy causes a collapse o the interbody space and instability will likely occur. Fig. 42-3 demonstrates a C6–C7 ACDF with the typical interposed gra t and plating system. Keyhole oraminotomy allows or decompressionwithoutrequiring usion,butitislesse ective
or removing centrally located canal pathology. (See Schwartz 10th ed., able 42-6 and Figure 42-27, pp. 1740–1741.)

68
Q
  1. A 35-year-old mother o two children, 5 and 6 years, has had amenorrhea and galactorrhea or the past 12 months. Her serum prolactin level is elevated, and radiographs o herskullshowan“emptysella.”T emostlikelydiagnosisis A. Menopause
    B. Pregnancy
    C. Pituitarytumor
    D. Sheehansyndrome
A

Answer:C
Pituitary adenomas arise rom the anterior pituitary gland (adenohypophysis). umors <1 cm diameter are considered microadenomas; larger tumors are macroadenomas. Pitu- itarytumorsmaybe unctional(ie,secreteendocrinologically active compounds at pathologic levels) or non unctional (ie, secrete nothing or inactive compounds). Functional tumors are o ten diagnosed when quite small, due to endocrine dys unction. he most common endocrine syndromes are Cushing disease, due to adrenocorticotropic hormone secretion, Forbes-Albright syndrome, due to prolactin secretion, and acromegaly, due to growth hormone secre- tion.Non unctionaltumorsaretypicallydiagnosedaslarger lesions causing mass e ects such as visual ield de icits due to compression o the optic chiasm or panhypopituitarism duetocompressiono thegland.Figure42-4demonstrates a large pituitary adenoma. Hemorrhage into a pituitary tumor causes abrupt symptoms o headache, visual distur- bance, decreased mental status, and endocrine dys unction.
his is known as pituitary apoplexy. (See Schwartz 10th ed., Figure 42-23, pp. 1735–1736.)

69
Q
11. T e most common intradural extramedullary tumors in children are
A. Ependymomas
B. Astrocytomas
C. Dermoids
D. eratomas
A

Answer:B
Astrocytomasarethemostcommonintramedullarytumorsin children, although they also occur in adults. hey may occur at all levels, although more o ten in the cervical cord. he tumor may inter ere with the CSF-containing central canal o the spinal cord, leading to a dilated central canal, re erred to as syringomyelia (syrinx). Spinal astrocytomas are usually low grade, but complete excision is rarely possible due to the nonencapsulated, in iltrative nature o the tumor. As a result, patients with astrocytomas are worse overall than patients with ependymomas. (See Schwartz 10th ed., pp. 1738–1739.)

70
Q
12. Diabetics, intravenous (IV) drug abusers, and dialysis patients have an increased risk o contracting the CNS
in ection
A. Pyogenic vertebral osteomyelitis
B. Subdural empyema
C. Discitis
D. Epidural abscess
A

Answer:A
Pyogenic vertebral osteomyelitis is a destructive bacterial in ection o the vertebrae, usually o the vertebral body. Ver- tebral osteomyelitis requently results rom hematogenous spread o distant disease, but may occur as an extension o adjacent disease, such as psoas abscess or perinephric abscess. Staphylococcus aureus and Enterobacter spp. are the most
requent etiologic organisms. Patients usually present with
ever and back pain. Diabetics, IV drug abusers, and dialysis patients have increased incidence o vertebral osteomyelitis. Epiduralextensionmayleadtocompressiono thespinalcord or nerve roots with resultant neurologic de icit. Osteomyelitis presents a lytic picture on imaging and must be distinguished
rom neoplastic disease. Adjacent intervertebral disc involve- mentoccurs requentlywithpyogenicosteomyelitis,butrarely with neoplasia. Plain ilms and computed tomography (C ) helpassesstheextento bonydestructionordeormitysuchas kyphosis. Magnetic resonance imaging (MRI) shows adjacent so t tissue or epidural disease. Most cases can be treated suc- cess ully with antibiotics alone, although the organism must be isolated to steer antibiotic choice. Blood cultures may be positive. Surgical intervention may be required or debride- ment when antibiotics alone ail, or or stabilization and
usioninthesettingo instabilityanddeormity.(SeeSchwartz 10th ed., p. 1745.)